1. Dirichlet's Theorem: Preliminary Results

Video. Dirichlet's Theorem Introduction

The main theorem of this unit will be the following:
Theorem
If \(d\) and \(r\) are relatively prime positive integers, then there are infinitely many primes \(p\) with \(p = q d + r\) for some integer \(q\) (i.e., infinitely many primes congruent to \(r\) modulo \(d\)).
The proof of the theorem will use quite a substantial portion of the tools that we have developed over the course of the year. It will be assumed that you are familiar with a number of basic facts about prime numbers. Perhaps the most sophisticated result we will take for grated is that every natural number \(N\) has a unique prime factorization \(N = p_1^{\ell_1} \cdots p_{n}^{\ell_n}\) where each exponent \(\ell_j\) is a nonnegative integer and the \(p_1,\ldots,p_n\) are distinct primes.

1.1. Factorization of the Riemann Zeta Function

Definition
The Riemann Zeta Function \(\zeta(s)\) is defined to equal
\[ \zeta(s) := \sum_{n=1}^\infty \frac{1}{n^s}\]
for each real \(s > 1\).
We know that the series does indeed converge when \(s > 1\) because it's simply a \(p\)-series. The Zeta function is intimately connected to the prime numbers. One particularly important way that this can be seen is through the following factorization theorem:
Theorem (Zeta Function Factorization)
For each \(s > 1\),
\[ \zeta(s) = \prod_{p \text{ prime}} \frac{1}{1 - p^{-s}}. \]
By the product over primes, we mean
\[ \prod_{p \text{ prime}} \frac{1}{1 - p^{-s}} := \lim_{N \rightarrow \infty} \prod_{\substack{p \text{ prime} \\ p \leq N}} \frac{1}{1 - p^{-s}}. \]
The proof of the Zeta function factorization is a consequence of the following lemma:
Lemma
For any positive integers \(K\) and \(N\), let \(p_1,\ldots,p_n\) be the prime numbers less than or equal to \(N\). Then for any \(s > 1\),
\[{}\prod_{\substack{p \text{ prime} \\ p \leq N}} \sum_{\ell=0}^K \frac{1}{(p^{\ell})^s}{}\]
\[{}= \sum_{\ell_1=0}^K \cdots \sum_{\ell_n=0}^K \frac{1}{(p_1^{\ell_1} \cdots p_n^{\ell_n})^s}{}\]
\[{}\leq \sum_{n=1}^\infty \frac{1}{n^s}.{}\]
Proof
The equality is simply a consequence of the distributive property; more precisely, it is easily proved by induction on the number of primes by summing the innermost sum, i.e.,
\[{}\sum_{\ell_1=0}^K \cdots \sum_{\ell_n=0}^K \frac{1}{(p_1^{\ell_1} \cdots p_n^{\ell_n})^s}{}\]
\[{}= \sum_{\ell_1=0}^K \cdots \sum_{\ell_{n-1}=0}^K \frac{1}{(p_1^{\ell_1} \cdots p_{n-1}^{\ell_{n-1}})^s}{}\]
\[{}\cdot \left( \sum_{\ell_n=0}^K \frac{1}{p_n^{\ell_ns}} \right){}\]
\[{}= \left( \sum_{\ell_1=0}^K \cdots \sum_{\ell_{n-1}=0}^K \frac{1}{(p_1^{\ell_1} \cdots p_{n-1}^{\ell_{n-1}})^s} \right){}\]
\[{}\cdot \left( \sum_{\ell_n=0}^K \frac{1}{p_n^{\ell_ns}} \right){}\]
\[{}= \left(\prod_{\substack{p \text{ prime} \\ p \leq p_{n}-1}} \sum_{\ell=0}^K \frac{1}{(p^{\ell})^s} \right){}\]
\[{}\cdot \left( \sum_{\ell_n=0}^K \frac{1}{p_n^{\ell_ns}} \right){}\]
\[{}= \prod_{\substack{p \text{ prime} \\ p \leq p_{n}}} \sum_{\ell=0}^K \frac{1}{(p^{\ell})^s}{}\]
\[{}= \prod_{\substack{p \text{ prime} \\ p \leq N}} \sum_{\ell=0}^K \frac{1}{(p^{\ell})^s},{}\]
where the last equality follows simply because there can be no more primes between \(p_n\) and \(N\).

To get the inequality, we use unique factorization: the map \((\ell_1,\ldots,\ell_n) \mapsto p_1^{\ell_1} \cdots p_n^{\ell_n}\) is injective because the prime factorization is unique. Thus if we let \(S_{N,K}\) be the set of natural numbers whose prime factorization involves only primes less than or equal to \(N\) with exponents \(\leq K\), it must be the case that
\[ \sum_{\ell_1=0}^K \cdots \sum_{\ell_n=0}^K \frac{1}{(p_1^{\ell_1} \cdots p_n^{\ell_n})^s} = \sum_{n \in S_{N,K}} \frac{1}{n^s}. \]
Because the terms are all nonnegative, if \(M\) is the largest element of \(S_{N,K}\), then
\[ \sum_{n \in S_{N,K}} \frac{1}{n^s} \leq \sum_{n=1}^M \frac{1}{n^s}, \]
and then the proof of the lemma follows by observing that the partial sums of a nonnegative series are dominated by the sum of the series.
Corollary 1
For any natural number \(N\) and any \(s > 1\),
\[ \prod_{\substack{p \text{ prime} \\ p \leq N}} \frac{1}{1-p^{-s}} \leq \sum_{n=1}^\infty \frac{1}{n^s}. \]
Proof
For each prime \(p\), since \(p > 1\), for any \(s > 0\) it must be the case that
\[ \frac{1}{1-p^{-s}} = \sum_{\ell=0}^\infty \frac{1}{(p^\ell)^s}, \]
and, in particular, the infinite series on the right-hand side must converge. Thus the corollary follows by fixing \(N\) and letting \(K \rightarrow \infty\) in the lemma above.
Corollary 2
For any natural number \(N\) and any \(s > 1\),
\[ \prod_{\substack{p \text{ prime} \\ p \leq N}} \frac{1}{1-p^{-s}} \geq \sum_{n=1}^N \frac{1}{n^s}. \]
Proof
We certainly must have
\[ \prod_{\substack{p \text{ prime} \\ p \leq N}} \sum_{\ell=0}^K \frac{1}{(p^{\ell})^s} \leq \prod_{\substack{p \text{ prime} \\ p \leq N}} \frac{1}{1-p^{-s}}\]
for each \(N\) and \(S\). By the Lemma, the product on the left-hand side is equal to the sum of \(n^{-s}\) as \(n\) ranges over natural numbers whose prime factorization involves primes less than \(N\) with powers less than \(K\). Fixing \(K = N\), because \(p^{\ell} \geq 2^{\ell} \geq \ell\) for any prime \(p\), every number \(n \leq N\) must itself have a prime factorization involving only primes less than or equal to \(N\) with powers less than or equal to \(N\), so \(\{1,\ldots,N\} \subset S_{N,N}\).
Proof (Proof of Zeta Function Factorization)
The proof follows immediately from Corollaries 1 and 2 by virtue of the Squeeze Theorem: Since
\[ \sum_{n=1}^N \frac{1}{n^s} \leq \prod_{\substack{p \text{ prime} \\ p \leq N}} \frac{1}{1-p^{-s}} \leq \sum_{n=1}^{\infty} \frac{1}{n^s}\]
and the left-hand side tends to the right-hand side as \(N \rightarrow \infty\), the product in the middle converges as \(N \rightarrow \infty\) and in the limit, all three “sides” must be equal.

2. A Theorem About the Primes

A sample of the kind of arguments we will use to prove Dirichlet's Theorem is the following result.
Theorem
\[ \sum_{p \text{ prime}} \frac{1}{p} = \infty. \]
Meta (The Meaning of the Sum)
Not only does this result imply that there are infinitely many primes, but It guarantees that the primes are not too sparse (e.g., not exponentially increasing).
Step 1 (Compare the Product to an Exponentiated Sum)
We have seen before that \(1 + x \leq e^x\) for all \(x \in {\mathbb R}\) by convexity of the exponential function. Fixing \(x = 1/(1-p^{-s}) - 1 = p^{-s}/(1-p^{-s})\) gives that
\[ \frac{1}{1-p^{-s}} \leq e^{\frac{p^{-s}}{1-p^{-s}}}\]
and so
\[ \prod_{\substack{p \text{ prime} \\ p \leq N}}\frac{1}{1-p^{-s}} \leq \exp \left( \sum_{\substack{p \text{ prime} \\ p \leq N}}\frac{p^{-s}}{1-p^{-s}} \right). \]
Letting \(N \rightarrow \infty\) gives that
\[ \zeta(s) \leq \exp \left( \sum_{\substack{p \text{ prime}}}\frac{p^{-s}}{1-p^{-s}} \right)\]
for each \(s > 1\).
Step 2 (Estimate the Zeta Function)
By the Integral Test,
\[ \zeta(s) \geq \int_1^\infty \frac{dx}{x^s} = \frac{1}{s-1}. \]
This means
\[ \ln \frac{1}{s-1} \leq \sum_{\substack{p \text{ prime}}}\frac{p^{-s}}{1-p^{-s}}\]
for each \(s > 1\).
Step 3 (Finishing the Proof)
When \(p\) is prime and \(s > 1\), \(p^{-s} \leq p^{-1} \leq 1/2\), so \(1-p^{-s} \geq 1/2\) and so
\[{}\ln \frac{1}{s-1}{}\]
\[{}\leq \sum_{\substack{p \text{ prime}}}\frac{p^{-s}}{1-p^{-s}}{}\]
\[{}\leq 2 \sum_{\substack{p \text{ prime}}} \frac{1}{p^s}{}\]
\[{}\leq 2 \sum_{p \text{ prime}} \frac{1}{p}.{}\]
Since \(\ln (s-1)^{-1} \rightarrow \infty\) as \(s \rightarrow 1^+\), the sum of \(1/p\) over all primes \(p\) cannot possibly be finite.